LSAT and Law School Admissions Forum

Get expert LSAT preparation and law school admissions advice from PowerScore Test Preparation.

User avatar
 Dave Killoran
PowerScore Staff
  • PowerScore Staff
  • Posts: 5850
  • Joined: Mar 25, 2011
|
#26560
Complete Question Explanation
(The complete setup for this game can be found here: lsat/viewtopic.php?t=11096)

The correct answer choice is (E)

If O is performed immediately after T, then O cannot be performed first, and O must be performed fifth. With T in fourth, the first rule comes into play, and we can infer that R must be performed third. When R is performed third, from the second rule we can infer that F must be performed sixth or seventh, leading to the following setup:
October 00_game#2_M12_L3_explanations_game#5_#15_diagram_1.png
Accordingly, answer choice (E) is correct.

Get the most out of your LSAT Prep Plus subscription.

Analyze and track your performance with our Testing and Analytics Package.